Is there a proof about angular momentum conservation?

In summary, the conversation discusses the conservation of angular momentum in a closed system and its proof through Noether's theorem and classical Newtonian mechanics. The exchange of angular momentum between objects in the system is constant, as demonstrated by the equal and opposite forces and momenta exerted on each other. This conservation can also be proven by the sum of torques being equal to the rate of change of angular momentum.
  • #1
LCSphysicist
646
161
Homework Statement
Angular momentum and it conservation.
Relevant Equations
L = rp
Angular momentum can be exchanged between objects in a closed system, but total angular momentum before and after an exchange remains constant (is conserved).
There is a proof about this conservation?
 
Physics news on Phys.org
  • #2
Consider two points in the system interacting, i.e. exerting a force on each other. If we accept the rule that action and reaction are equal and opposite then these forces are equal and opposite, ##\vec F## and ##-\vec F##, and lie in the same line.
If the forces exist for time dt then the points impart equal and opposite momenta, ##\vec Fdt## and ##-\vec Fdt##.
If we consider all this in respect of some axis O, and a vector from there to the line of action of the forces is ##\vec r##, then the angular momenta they impart on each other are ##\vec r\times\vec Fdt## and ##-\vec r\times\vec Fdt##.
Hence the sum of the imparted angular momenta is zero.
 
  • Like
Likes LCSphysicist, archaic and FactChecker
  • #3
  • Like
Likes Delta2
  • #4
ORF said:
Hi,

A more abstract (and fancy) proof is given by Noether's theorem: if your system is invariant under rotation, angular momentum is conserved

https://en.wikipedia.org/wiki/Noether's_theorem#Applications

Regards,
ORF
Wow, this was scarry, i was just reading this in French Newtonian mechanics right now about this, like ten minutes ago kkkk unfortunately I don't understand much about these concepts of symmetry. but i will save it thx
 
  • #5
haruspex said:
Consider two points in the system interacting, i.e. exerting a force on each other. If we accept the rule that action and reaction are equal and opposite then these forces are equal and opposite, ##\vec F## and ##-\vec F##, and lie in the same line.
If the forces exist for time dt then the points impart equal and opposite momenta, ##\vec Fdt## and ##-\vec Fdt##.
If we consider all this in respect of some axis O, and a vector from there to the line of action of the forces is ##\vec r##, then the angular momenta they impart on each other are ##\vec r\times\vec Fdt## and ##-\vec r\times\vec Fdt##.
Hence the sum of the imparted angular momenta is zero.
You are using a theorem of classical Newtonian mechanics here, namely that the sum of torques ##\sum \vec{T_i}=\frac{d\vec{L}}{dt}## equals the rate of change of angular momentum L. It is a theorem in the sense that it can be proved from Newton's laws.
 
  • #6
Delta2 said:
You are using a theorem of classical Newtonian mechanics here, namely that the sum of torques ##\sum \vec{T_i}=\frac{d\vec{L}}{dt}## equals the rate of change of angular momentum L. It is a theorem in the sense that it can be proved from Newton's laws.
Start from Newton's second law ##\vec F = m \vec a ##.
By definition, the angular momentum is ##\vec L = \vec r \times\vec p= m \vec r \times\vec v##. (1)
(x means cross product)
## \vec r ## is the position vector, ## \vec v ## is the velocity ## \vec v = \dot {\vec r}##
##\vec a ## is the acceleration,##\dot{\vec v}= \vec a##.
The force is ##\vec F =m\vec a## .
The torque is ##\vec\tau=\vec r \times \vec F##
Take the time derivative of eq. (1)
##\dot{\vec L }= m \left (\dot{\vec r}\times \vec v + \vec r \times \dot{\vec v}\right)##
##\dot{\vec r}\times \vec v ==0##
so
##\dot{\vec L }= m \vec r \times \dot{\vec v}= m \vec r \times \vec a = \vec r \times\vec F= \vec \tau##
 
Last edited:
  • Like
Likes Delta2

Related to Is there a proof about angular momentum conservation?

1. What is angular momentum conservation?

Angular momentum conservation is a fundamental law of physics that states that the total angular momentum of a system remains constant in the absence of external torques. In simpler terms, it means that the rotational motion of an object will remain the same unless acted upon by an external force.

2. Is there a proof for angular momentum conservation?

Yes, there is a mathematical proof for angular momentum conservation. It is derived from the laws of motion and the principles of rotational dynamics. The proof shows that the total angular momentum of a system is equal to the sum of the individual angular momenta of its components.

3. How is angular momentum conserved?

Angular momentum is conserved due to the law of conservation of angular momentum, which states that the total angular momentum of a system remains constant in the absence of external torques. This means that any changes in the angular momentum of one component of the system will be balanced out by an equal and opposite change in the angular momentum of another component.

4. Can angular momentum conservation be violated?

No, angular momentum conservation cannot be violated. It is a fundamental law of physics that has been extensively tested and has always been found to hold true. However, in certain cases, it may appear that angular momentum is not conserved due to external forces or torques acting on the system, but the total angular momentum will always remain constant.

5. Why is angular momentum conservation important?

Angular momentum conservation is important because it is a fundamental law of physics that helps us understand and predict the behavior of rotating systems. It is also crucial in many practical applications, such as spacecraft navigation, gyroscopes, and the motion of planets and stars. Additionally, it is a key principle in the conservation of energy, which is a fundamental concept in physics.

Similar threads

  • Introductory Physics Homework Help
Replies
17
Views
161
  • Introductory Physics Homework Help
Replies
2
Views
832
  • Introductory Physics Homework Help
Replies
5
Views
152
  • Introductory Physics Homework Help
2
Replies
57
Views
1K
  • Introductory Physics Homework Help
Replies
4
Views
1K
  • Introductory Physics Homework Help
Replies
23
Views
952
  • Introductory Physics Homework Help
Replies
23
Views
1K
  • Introductory Physics Homework Help
Replies
30
Views
2K
  • Introductory Physics Homework Help
Replies
4
Views
864
Back
Top